L ' Hopital's Rule for x -> infinity

  • Thread starter unscientific
  • Start date
  • Tags
    Infinity
In summary, L'Hopital's rule can be applied for limits approaching infinity in the same way as for limits approaching 0, by taking the reciprocal of the functions and using the derivative of the resulting functions in the limit calculation.
  • #1
unscientific
1,734
13

Homework Statement


Hopital's Rule for x -> ∞ applies the same way as x -> 0.


Homework Equations



As shown in attached pic.

The Attempt at a Solution



I tried to prove that x->∞ works the same way as x -> 0, only to get its reciprocal.. Not sure what is wrong with my working..
 

Attachments

  • hopital2.jpg
    hopital2.jpg
    9.4 KB · Views: 399
  • Hopital.jpg
    Hopital.jpg
    13.9 KB · Views: 424
Physics news on Phys.org
  • #2
unscientific said:

Homework Statement


Hopital's Rule for x -> ∞ applies the same way as x -> 0.

Homework Equations



As shown in attached pic.

The Attempt at a Solution



I tried to prove that x->∞ works the same way as x -> 0, only to get its reciprocal.. Not sure what is wrong with my working..

Why should h or k have a derivative at a? You'll have to come up with different reasoning.
 
  • #3
You ask about L'Hopital's rule applying "as x-> infinity" but your attachment has x->a while it is f(x) and g(x) that go to infinity. Which are you trying to prove?
 
  • #4
HallsofIvy said:
You ask about L'Hopital's rule applying "as x-> infinity" but your attachment has x->a while it is f(x) and g(x) that go to infinity. Which are you trying to prove?

No only that, you applied Taylor expansions of f(x) and g(x) at x = a, where both functions are infinite! You can't do that.

RGV
 
  • #5
HallsofIvy said:
You ask about L'Hopital's rule applying "as x-> infinity" but your attachment has x->a while it is f(x) and g(x) that go to infinity. Which are you trying to prove?

Sorry for the confusion! I mean f(a) = g(a) = ∞

I earlier showed that l'Hopital's Rule works when its 0/0 but I am now trying to show it works for ∞/∞.

So i took (1/g)/(1/f) to make it 0/0..

But when i use taylor's expansion something's wrong..
 
  • #6
unscientific said:
Sorry for the confusion! I mean f(a) = g(a) = ∞

I earlier showed that l'Hopital's Rule works when its 0/0 but I am now trying to show it works for ∞/∞.

So i took (1/g)/(1/f) to make it 0/0..

But when i use taylor's expansion something's wrong..


What is Taylor's expansion for 1/(f(x)) , expanded about x = a ?
This poses a problem since 1/f(a) is undefined.​

Construct two functions, F(x) & G(x) such that F(a) = 0 = G(a), otherwise F(x) = 1/f(x) and G(x) = 1/g(x) .
 
  • #7
SammyS said:
What is Taylor's expansion for 1/(f(x)) , expanded about x = a ?
This poses a problem since 1/f(a) is undefined.​

Construct two functions, F(x) & G(x) such that F(a) = 0 = G(a), otherwise F(x) = 1/f(x) and G(x) = 1/g(x) .

Yes i did that, i let 1/g = h, 1/f = k.

then it reduces to [ h'(x)/k'(x) ] which translates to [g'(x)/f'(x)], which is the reciprocal instead..
 
  • #8
unscientific said:
Yes i did that, i let 1/g = h, 1/f = k.

then it reduces to [ h'(x)/k'(x) ] which translates to [g'(x)/f'(x)], which is the reciprocal instead..

Check that again.

So, [itex]\displaystyle h(x)=\frac{1}{g(x)}\quad\to\quad h'(x)=-\frac{g'(x)}{(g(x))^2}\ .[/itex]

A similar result holds for k'(x) .

[itex]\displaystyle \lim_{x\to a}\frac{f(x)}{g(x)}=\lim_{x\to a}\frac{h(x)}{k(x)}[/itex]
[itex]\displaystyle =\lim_{x\to a}\frac{h'(x)}{k'(x)}[/itex]

[itex]\displaystyle =\lim_{x\to a}\frac{g'(x)}{(g(x))^2}\frac{(f(x))^2}{f'(x)}[/itex]

[itex]\displaystyle =\lim_{x\to a}\frac{g'(x)}{f'(x)}\ \lim_{x\to a}\frac{(f(x))^2}{(g(x))^2}[/itex]​

Can you take it from there?
 
  • #9
SammyS said:
Check that again.

So, [itex]\displaystyle h(x)=\frac{1}{g(x)}\quad\to\quad h'(x)=-\frac{g'(x)}{(g(x))^2}\ .[/itex]

A similar result holds for k'(x) .

[itex]\displaystyle \lim_{x\to a}\frac{f(x)}{g(x)}=\lim_{x\to a}\frac{h(x)}{k(x)}[/itex]
[itex]\displaystyle =\lim_{x\to a}\frac{h'(x)}{k'(x)}[/itex]

[itex]\displaystyle =\lim_{x\to a}\frac{g'(x)}{(g(x))^2}\frac{(f(x))^2}{f'(x)}[/itex]

[itex]\displaystyle =\lim_{x\to a}\frac{g'(x)}{f'(x)}\ \lim_{x\to a}\frac{(f(x))^2}{(g(x))^2}[/itex]​

Can you take it from there?

Yes I ended up with that step. So f(a) = g(a), f/g = 1.

Then we have g'(x)/f'(x), which is the reciprocal, where I am stuck at...
 
  • #10
unscientific said:
Yes I ended up with that step. So f(a) = g(a), f/g = 1.

Then we have g'(x)/f'(x), which is the reciprocal, where I am stuck at...
What is that reciprocal equal to ?
 
  • #11
SammyS said:
What is that reciprocal equal to ?

By right I should get f'(x)/g'(x) as when approximating x→0.

But I'm getting g'(x)/f'(x) instead..
 
  • #12
SammyS said:
[itex]\displaystyle \lim_{x\to a}\frac{f(x)}{g(x)}=\dots[/itex]
[itex]\displaystyle =\lim_{x\to a}\frac{g'(x)}{f'(x)}\ \lim_{x\to a}\frac{(f(x))^2}{(g(x))^2}[/itex]​

unscientific said:
By right I should get f'(x)/g'(x) as when approximating x→0.

But I'm getting g'(x)/f'(x) instead..
Suppose both limits [itex]\displaystyle \lim_{x\to a}f'(x) \,,\ \lim_{x\to a}g'(x)[/itex] exist & that [itex]\displaystyle \lim_{x\to a}f'(x)\ne0\ .[/itex]

Then the following should give the desired result.
[itex]\displaystyle \lim_{x\to a}\frac{f(x)}{g(x)}=
\frac{\lim_{x\to a}g'(x)}{\lim_{x\to a}f'(x)}\cdot \lim_{x\to a}\frac{f(x)}{g(x)}\cdot \lim_{x\to a}\frac{f(x)}{g(x)}
[/itex]​
 
  • #13
SammyS said:
Suppose both limits [itex]\displaystyle \lim_{x\to a}f'(x) \,,\ \lim_{x\to a}g'(x)[/itex] exist & that [itex]\displaystyle \lim_{x\to a}f'(x)\ne0\ .[/itex]

Then the following should give the desired result.
[itex]\displaystyle \lim_{x\to a}\frac{f(x)}{g(x)}=
\frac{\lim_{x\to a}g'(x)}{\lim_{x\to a}f'(x)}\cdot \lim_{x\to a}\frac{f(x)}{g(x)}\cdot \lim_{x\to a}\frac{f(x)}{g(x)}
[/itex]​

I see! then the f(x)/g(x) cancels on both sides leaving only one f(x)/g(x) on the right, then you bring over the g'(x)/f'(x) to the left to give f'(x)/g'(x) as desired!
 

1. What is L'Hopital's Rule?

L'Hopital's Rule is a mathematical theorem that allows us to find the limit of a function as it approaches infinity or negative infinity, by using the derivatives of the numerator and denominator of the function.

2. When can L'Hopital's Rule be applied?

L'Hopital's Rule can be applied when we have an indeterminate form, such as 0/0 or ∞/∞, when taking the limit of a function as x approaches infinity.

3. How do we use L'Hopital's Rule?

To use L'Hopital's Rule, we first take the derivative of the numerator and denominator of the function. Then, we evaluate the limit of this new function. If the limit still results in an indeterminate form, we can repeat the process until we get a definitive answer or until it is no longer an indeterminate form.

4. Can L'Hopital's Rule be used for any type of function?

No, L'Hopital's Rule can only be used for functions that are continuous and differentiable in the interval where the limit is being evaluated. It also only applies to limits as x approaches infinity or negative infinity.

5. Are there any limitations to L'Hopital's Rule?

Yes, L'Hopital's Rule may not work if the function does not have a finite limit as x approaches infinity or negative infinity, or if the limit is not an indeterminate form. In these cases, other methods must be used to evaluate the limit.

Similar threads

Replies
1
Views
630
  • Calculus and Beyond Homework Help
Replies
3
Views
2K
  • Calculus and Beyond Homework Help
Replies
4
Views
984
  • Calculus and Beyond Homework Help
Replies
5
Views
1K
Replies
3
Views
872
  • Calculus and Beyond Homework Help
Replies
7
Views
706
  • Calculus and Beyond Homework Help
Replies
25
Views
348
  • Calculus and Beyond Homework Help
Replies
2
Views
2K
  • Calculus and Beyond Homework Help
Replies
14
Views
2K
  • Calculus and Beyond Homework Help
Replies
2
Views
899
Back
Top